Quadratic Inequalities

Quadratic Inequalities

Answers

Answer 1

The complete table of values is

x   1    1.5     2     3    3.5     4     5

y  1.33 -1.58 -2.17 -1.33 -0.43 0.71 3.57

The graph is attachedThe x values are {1.28, 4.76}The x values are undefined The x values are {1.15, 3.69}

Completing the table of values

The equation of the function is given as

y = x²/3 + 6/x² - 5

To complete the table of values, we set x = 1, 1.5, 4 and 5

So, we have

y = 1²/3 + 6/1² - 5 = 1.33

y = 1.5²/3 + 6/(1.5²) - 5 = -1.58

y = 4²/3 + 6/(4²) - 5 = 0.71

y = 5²/3 + 6/(5²) - 5 = 3.57

Solving the x values from the graph

The x and the y intervals are given as

0 ≤ x ≤ 5 and -5 ≤ y ≤ 4

See attachment for the graph and the labelled points

Estimating x²/3 + 6/x² - x - 3 = 0

We have

y = x²/3 + 6/x² - 5

Set y = x - 2

x²/3 + 6/x² - 5 = x - 2

So, we have

x²/3 + 6/x² - x - 3 = 0

This means that y = x - 2

From the graph, we have x = {1.28, 4.76}

Estimating x²/3 + 6/x² - x = 0

We have

y = x²/3 + 6/x² - 5

Set y = x - 5

x²/3 + 6/x² - 5 = x - 5

So, we have

x²/3 + 6/x² - x = 0

This means that y = x - 5

From the graph, we have x = undefined

It has no solution because the line does not intersect with the curve

Estimating x²/3 + 6/x² - 5 = 0

We have

y = x²/3 + 6/x² - 5

This means that y = 0

From the graph, we have x = {1.15, 3.69}

Read more about functions at

https://brainly.com/question/27915724

#SPJ1

Quadratic Inequalities

Related Questions

Josie had 12 beads on her necklace. Then she added b more beads. Write an expression that shows how many beads are on the necklace in all. please write a expression like b-12 or b+?

Answers

The expression that shows how many beads are on the necklace in all is:

b + 12.

What is expression?

An expression in mathematics is a combination of numbers, variables, and/or operators that represents a mathematical relationship or quantity. It may contain constants, variables, coefficients, and mathematical operations such as addition, subtraction, multiplication, division, and exponentiation. Expressions are often used to describe or represent real-world situations, and can be simplified, evaluated, or manipulated using algebraic rules and properties.

In the given question,

To write an expression that shows how many beads are on Josie's necklace in all after adding b more beads:

Start with the initial number of beads on the necklace, which is 12.

To this, add the number of beads Josie added, which is b.

Combine the two terms to form the final expression: 12 + b.

Therefore, the expression that shows how many beads are on Josie's necklace in all after adding b more beads is 12 + b.

To know more about expressions and equations, visit:

https://brainly.com/question/11537309

#SPJ1

The vertices of a quadrilateral in the coordinate plane are known. How can the perimeter of the figure be found?
O Use the distance formula to find the length of each side, and then add the lengths.
Use the slope formula to find the slope of each of side, and then determine if the opposite sides are parallel.
O Use the slope formula to find the slope of each of side, and then determine if the consecutive sides are perpendicul
O Use the distance formula to find the length of the sides, and then multiply two of the side lengths.

Answers

Answer:..

Step-by-step explanation:Use the distance formula to find the length of each side and then add the lengths.

Use the slope formula to find the slope of each of side, and then determine if the opposite sides are parallel.

Use the slope formula to find the slope of each of side, and then determine if the consecutive sides are perpendicular.

Use the distance formula to find the length of the sides, and then multiply two of the side lengths.

Ivan created a scale drawing of the Grand Canyon using a scale of 1 inch for every 25
miles. His drawing is 11 inches long. What is the actual length of the canyon?

Answers

Answer:

275 miles

Step-by-step explanation:

A cuboid is placed on top of a cube, as shown in the diagram, to form a solid.
2 cm
3 cm
The cube has edges of length 7 cm.
The cuboid has dimensions 2 cm by 3 cm by 5 cm.
Work out the total surface area of the solid.
Optional working
Ansv
cm²
+
5 cm
7 cm

Answers

Answer: 344cm²

Step-by-step explanation:

7x7=49

49x5=245

3x2=6

49-6=43

245+43=288

5x2=10 10x2=20

5x3=15 15x2=30

288+30+20+6=344

Victoria has $200 of her birthday gift money saved at home, and the amount is modeled by the function h(x) = 200. She reads about a bank that has savings accounts that accrue interest according to the function s(x) = (1. 05)x−1. Explain how Victoria can combine the two functions to model the total amount of money she will have in her bank account as interest accrues after she deposits her $200. Justify your reasoning.


WILL GIVE BRANLIEST

Answers

The reasoning behind this is that the initial amount (h(x)) is multiplied by the interest growth factor (s(x)) to calculate the total amount with interest over time.

To model the total amount of money Victoria will have in her bank account as interest accrues after depositing her $200, you can combine the two functions h(x) and s(x). The given functions are h(x) = 200 and s(x) = (1.05)^x−1.

First, note that s(x) represents the growth factor of the interest, which is 5% (1.05) compounded annually. To find the total amount after x years, you need to multiply the initial amount by the growth factor raised to the power of x.

So, the combined function T(x) can be written as T(x) = h(x) * s(x).

Substitute the given functions into the combined function:

T(x) = (200) * ((1.05)^x−1)

This function, T(x), models the total amount of money Victoria will have in her bank account after x years with interest accrued on her $200 deposit. The reasoning behind this is that the initial amount (h(x)) is multiplied by the interest growth factor (s(x)) to calculate the total amount with interest over time.

To learn more about Interest

https://brainly.com/question/30819806

#SPJ11

Casey is going to wear a gray sportscoat and is trying to decide what tie he should wear to work. In his​ closet, he has 22 ​ties, 13 of which he feels go well with the sportscoat. If Casey selects one tie at​ random, determine the probability and the odds of the tie going well or not going well with the sportscoat.

The probability the tie goes well with the jacket is?

The probability the tie will not go well with the jacket is?

The odds against the tie going well with the jacket is?

(Simplify your answer. Type an integer or a​ fraction.)

The odds in favor of the tie going well with the jacket is? ​

(Simplify your answer. Type a ratio using a​ colon.)

Answers

The probability that the tie goes well with the jacket is:

P(going well) = number of ties that go well / total number of ties
P(going well) = 13/22
P(going well) = 0.59 or 59%

The probability that the tie will not go well with the jacket is:

P(not going well) = 1 - P(going well)
P(not going well) = 1 - 0.59
P(not going well) = 0.41 or 41%

The odds against the tie going well with the jacket is:

Odds against = number of ties that do not go well / number of ties that go well
Odds against = (22-13) / 13
Odds against = 9/13

The odds in favor of the tie going well with the jacket is:

Odds in favor = number of ties that go well / number of ties that do not go well
Odds in favor = 13 / (22-13)
Odds in favor = 13/9 or 1.44:1

(If this doesn’t seem right to you comment!)

John owns 10 shirts and needs to select 5 to wear to school next week. How many ways can he choose the shirts? (hint: combinations)
50 ways
B 252 ways
500 ways
D 792 ways

Answers

There are 252 different combinations of shirts, the correct option is B.

In how many ways he can choose the shirts?

Remember that if you have N elements, and you want to select K of these, the number of combinations is given by:

[tex]C(N, K) = \frac{N!}{(N - K)!*K!}[/tex]

Here we have 10 shirts and we want to select 5 of these, so the number of combinations is given by:

[tex]C(10, 5) = \frac{10!}{(10 - 5)!*5!} \\\\C(10, 5) = \frac{10*9*8*7*6}{5*4*3*2} = 252[/tex]

There are 252 different combinations, so the correct option is B.

Learn more about combinations at:

https://brainly.com/question/11732255

#SPJ1

A 2-quart carton of pineapple juice costs $8.08. What is the price per cup?

$

Answers

Answer:

$1.01

Step-by-step explanation:

We Know

A 2-quart carton of pineapple juice costs $8.08

1 quart = 4 cups

2 quarts = 8 cups

So, 8 cups of pineapple juice cost $8.08.

What is the price per cup?

We Take

8.08 / 8 = $1.01

So, the price per cup is $1.01

The sum of the roots of a quadratic is 1 and the product of the roots is -35/4.

a. find the quadratic.

b. find the roots

Answers

If the sum of the roots of a quadratic equation is 1 and the product of the roots is -35/4 and the equation is [tex]4x^2-4x-35=0[/tex] and the roots are 3.5 and -2.5

If the quadratic equation is [tex]ax^2+bx+c=0[/tex]

The sum of the roots = [tex]-\frac{b}{a}[/tex]

The product of the roots = [tex]\frac{c}{a}[/tex]

Sum of the roots = 1 = [tex]-\frac{b}{a}[/tex]

Product of the roots = [tex]-\frac{35}{4}[/tex] = [tex]\frac{c}{a}[/tex]

If we assume a as 1, then the equation comes out to be:

[tex]x^2-x-\frac{35}{4} =0[/tex]

Multiply the equation by 4 to get a simplified equation:

[tex]4x^2-4x-35=0[/tex]

[tex]4x^2[/tex] - 14x + 10x - 35 = 0

2x (2x - 7) + 5 (2x - 7) = 0

(2x - 7)(2x + 5) = 0

x = 3.5 and -2.5

Learn more about equation:

https://brainly.com/question/28871326

#SPJ4

stressing outtt I need help its due in a few minuets

Answers

First, we can simplify the expression 64e^2/5e by dividing the numerator by the denominator. Since e is a common factor, we can cancel it out:
64e^2/5e = 64e^(2-1)/5 = 64e/5

Similarly, we can simplify the expression 3e/8e by dividing the numerator by the denominator and canceling out e:
3e/8e = 3/8

Now we can multiply the two simplified expressions to get the final answer:
(64e/5) * (3/8) = (64*3*e)/(5*8) = 24.96e

Therefore, the simplified expression is 24.96e.

Greg, Harry and Ian share their electricity bill in the ratio 2:4:5.
how much dies each of them pay when their electricity bill are 1) 110$ 2) 165$ 3) 352$
pls answer quickly​

Answers

The amount each of them pays when their electricity bill is $110, $165, and $352 respectively, in the ratio 2:4:5, are:

1) $20, $40, $50

2) $30, $60, $75

3) $64, $128, $160

1. How much do they pay for a $110 electricity bill?

To find out how much each of them pays, we'll use the given ratio of 2:4:5 and divide the total bill among them accordingly.

Total bill: $110

The total ratio is 2+4+5=11.

Greg's share: (2/11) * $110 = $20

Harry's share: (4/11) * $110 = $40

Ian's share: (5/11) * $110 = $50

Therefore, Greg pays $20, Harry pays $40, and Ian pays $50.

2. How much do they pay for a $165 electricity bill?

Total bill: $165

The total ratio is still 2+4+5=11.

Greg's share: (2/11) * $165 = $30

Harry's share: (4/11) * $165 = $60

Ian's share: (5/11) * $165 = $75

Therefore, Greg pays $30, Harry pays $60, and Ian pays $75.

3. How much do they pay for a $352 electricity bill?

Total bill: $352

The total ratio remains the same: 2+4+5=11.

Greg's share: (2/11) * $352 = $64

Harry's share: (4/11) * $352 = $128

Ian's share: (5/11) * $352 = $160

Therefore, Greg pays $64, Harry pays $128, and Ian pays $160.

Learn more about ratio

brainly.com/question/13419413

#SPJ11

What is the mean of the data set fifth grade jump distance

Answers

The mean of the fifth-grade jump distance data set.

How to calculate the mean of fifth-grade jump distances?

To determine the mean of the data set for fifth-grade jump distances, we need the actual data values. Without the specific data set, it is not possible to calculate the mean.

The mean is the sum of all the values in a data set divided by the number of values. Therefore, we would need the individual jump distances for each fifth-grade student to calculate the mean accurately.

Once we have the complete data set, we can add up all the distances and divide by the total number of students to find the mean. Without the specific data, we cannot provide a numerical answer for the mean of the fifth-grade jump distance.

Learn more about mean
brainly.com/question/521501
#SPJ11

Graph the equation shown below by transforming the given graph of the parent function. 2^3x ​

Answers

The graph of the exponential function [tex]y = 2^{3x}[/tex] is given as follows:

How to define an exponential function?

An exponential function has the definition presented as follows:

[tex]y = ab^x[/tex]

In which the parameters are given as follows:

a is the value of y when x = 0.b is the rate of change.

The function in this problem is given as follows:

[tex]y = 2^{3x}[/tex]

Hence the parameter values are:

a = 1, hence when x = 0, y = 1.b = 2, hence when x increases by 1/3, y is multiplied by 2 -> increases by 1/3 due to the horizontal compression with the multiplication by 3 in the domain.

More can be learned about exponential functions at brainly.com/question/2456547

#SPJ1

Question 1 2 pts A researcher is interested in determining whether sugar consumption increases memory. She recruits 100 participants and randomly assigns them to one of two groups, sugar vs. No sugar. She then measures their word recall: Sugar group: M = 4. 5, SD = 1. 8, n = 55 • No sugar group: M = 2. 6, SD =. 68, n = 45 1. Is this an example of a quasi-experimental design? (yes or no) 2. What is the dependent variable? 3. What is the independent variable?​

Answers

1. This is not an example of a quasi-experimental design because the researcher randomly assigned participants to the two groups. In a quasi-experimental design, the researcher does not have full control over the assignment of participants to groups.

2. The dependent variable in this study is the word recall. It is the variable that the researcher is interested in measuring and is expected to be affected by the independent variable.

3. The independent variable in this study is sugar consumption. It is the variable that the researcher manipulated by assigning participants to one of two groups, sugar vs. no sugar.

The purpose of manipulating the independent variable is to examine its effect on the dependent variable, word recall.

To learn more about variable, refer below:

https://brainly.com/question/17344045

#SPJ11

Determine the location and value of the absolute extreme values off on the given interval, if they exist 8x? f(x) - +22x2 - 24x on (-7,11 G What in are the absolute maximum maxime off on the given interval? Select the correct choice below and, if necessary, to in the answer bowen to completo your choice A. Tho absolute maximum/maxima isarea- (Use a comma to separato answers as needed. Type exact answers, using radicals as rended) B. There is no absolute maximum off on the given interval What are the absolute minimum/minima off on the given interval? Select the correct choice below and. If necessary, in the wwer boxes to complete your in O A The absolute minimum/minima is/are at (Use a comma to separate answers as needed. Type exact answers, using radical as needed) B. There is no absolute minimum off on the given interval

Answers

The absolute maximum will be at (11, 10373).
The absolute minimum is at (-1.963, -25.294).

What in are the absolute maximum maxime off on the given interval?

To determine the location and value of the absolute extreme values of the function f(x) = 8x³+ 22x² - 24x on the interval (-7, 11), follow these steps:

Find the critical points by taking the derivative of the function and setting it equal to zero:
f'(x) = 24x² + 44x - 24

Solve for x:
Factor the equation: 4(6x² + 11x - 6) = 0
Using the quadratic formula, x = (-11 ± √(121 + 144))/12
x ≈ -1.963, 0.630

Check the endpoints and the critical points to find the absolute maximum and minimum:
f(-7) ≈ 461
f(-1.963) ≈ -25.294
f(0.630) ≈ -16.102
f(11) ≈ 10373

Compare the values:
The absolute maximum is at x = 11, with a value of 10373.
The absolute minimum is at x ≈ -1.963, with a value of ≈ -25.294.

Answer:
The absolute maximum is at (11, 10373).
The absolute minimum is at (-1.963, -25.294).

Learn more about critical points.

brainly.com/question/31017064

#SPJ11

An office manager orders one calculator or one calendar for each of the office's 80 employees. Each calculator costs $12, and each calendar costs $10. The entire order totaled $900.
Part A: Write the system of equations that models this scenario.
Part B: Use substitution method or elimination method to determine the number of calculators and calendars ordered. Show all necessary steps.

Answers

The system of equations is.

x + y = 80

12x + 10y = 900

And the solutions are y = 30 and x = 50

How to write and solve the system of equations?

Let's define the two variables:

x = number of calculators.

y = number of calendars.

With the given information we can write two equations, then the system will be:

x + y = 80

12x + 10y = 900

Now let's solve it.

We can isolate x on the first equation to get:

x = 80 - y

Replace that in the other equation to get:

12*(80 - y) + 10y = 900

-2y = 900 - 960

-2y = -60

y = -60/-2 = 30

Then x = 50

Learn more about systems of equations at:

https://brainly.com/question/13729904

#SPJ1

In the year 2001, a company made $7. 2 million in profit. For each consecutive year after that, their profit increased by 11%. How much would the company's profit be in the year 2005, to the nearest tenth of a million dollars?​

Answers

The company's profit in the year 2005 would be approximately $10.5 million.

Let's calculate the company's profit for the year 2005 using the given information.

Initial profit in 2001: $7.2 million
Annual profit increase: 11%

We need to find the profit for 2005, which is 4 years after 2001.

Step 1: Identify the formula for compound interest, which can be applied to profit increase:
Future Profit = Initial Profit * (1 + Profit Increase Rate)^Number of Years

Step 2: Plug in the values:
Future Profit [tex]= $7.2 million * (1.11)^4[/tex]
Step 3: Calculate the result:
Future Profit [tex]= $7.2 million * (1.11)^4[/tex]
Future Profit = $7.2 million [tex]* 1.4641[/tex]
Future Profit = $10.54152 million

Step 4: Round the result to the nearest tenth of a million dollars:
Future Profit ≈ $10.5 million

So, the company's profit in the year 2005 would be approximately $10.5 million.

Learn more about compound interest,

https://brainly.com/question/28020457

#SPJ11

The number of revolutions
made by a tire traveling over a fixed distance
varies inversely with the radius of the tire. a
12-inch radius tire makes 100 revolutions to
travel a certain distance. how many
revolutions would a 16-inch radius tire require
a
to travel the same distance?

define variables
identify constant of variation
write an equation and show work (please can someone help with this stuff, my deadline is in 4 days)

Answers

A 16-inch radius tire would require 75 revolutions to travel the same distance as a 12-inch radius tire that made 100 revolutions.

Let's start by defining some variables. Let "r" represent the radius of the tire and "n" represent the number of revolutions it makes over a fixed distance. We are given that the number of revolutions is inversely proportional to the radius of the tire. This means that as the radius increases, the number of revolutions decreases, and vice versa. We can express this relationship mathematically as follows:

n = k/r

Here, "k" is the constant of variation, which remains the same for any given tire traveling over the same distance. To solve the problem, we need to find the value of "k" first. We know that a 12-inch radius tire makes 100 revolutions to travel a certain distance. Substituting these values into the equation, we get:

100 = k/12

Solving for "k," we get k = 1200. Now we can use this value to find the number of revolutions required by a 16-inch radius tire to travel the same distance:

n = 1200/16 = 75

Therefore, a 16-inch radius tire would require 75 revolutions to travel the same distance as a 12-inch radius tire that made 100 revolutions.

In summary, we defined the variables, identified the constant of variation, wrote the equation (n=k/r), found the value of the constant by using the given information, and used it to solve the problem by finding the number of revolutions required by a tire with a different radius.

Know more about revolutions here:

https://brainly.com/question/20700248

#SPJ11

Lily's age is 2 years and 4 months.
Hugo's age is 1 year and 8 months.
Write Lily's age in months as a fraction of Hugo's age in months.
Give your answer in it's simplest form.

Answers

Answer:

7/5

Step-by-step explanation:

Lily's age in months: 2×12+4=28mo

Hugo's age in months=12+8=20

Lily's age over Hugo's age: 28/20

Simplify:

28/20=14/10=7/5

11. The spread of a drop of food dye throughout a glass of milk is called__________

Answers

Answer: Diffusion

Step-by-step explanation:

(4,7);y=3x+6
Write an equation passing through the point and parallel to the given line.

Answers

The Equation of the line which is parallel to the line "y = 3x + 6", and also passes through (4,7) is "y = 3x - 5".

In order to find an equation of a line which passes through point (4,7) and is parallel to line "y = 3x + 6", we use the fact that parallel lines have the same slope.

The given line ""y = 3x + 6" has a slope of 3,

So, the "parallel-line" we want to find must also have a slope of 3.

Now, by using the "point-slope" form of the equation of a line, which is : y - y₁ = m(x - x₁),

where m is slope and (x₁, y₁) is a point on the line,

So, we substitute "m = 3" and (x₁, y₁) = (4,7) to get:

⇒ y - 7 = 3(x - 4),

⇒ y - 7 = 3x - 12,

⇒ y = 3x - 5

Therefore, the equation of the required line is y = 3x - 5.

Learn more about Equation Of Line here

https://brainly.com/question/25564723

#SPJ1

A hypothesis regarding the weight of newborn infants at a community hospital is that the mean is 19. 1 pounds. A sample of seven infants is randomly selected and their weights at birth are recorded as 18. 1, 21. 1, 22. 1, 23. 1, 21. 1, 27. 1, and 27. 1 pounds. If α = 0. 200, what is the critical value? The population standard deviation is unknown

Answers

Since the population standard deviation is unknown, we use a t-distribution to find the critical value. The degrees of freedom for the t-distribution is n-1, where n is the sample size. In this case, n = 7, so the degrees of freedom is 7-1 = 6. The critical value for a t-distribution with 6 degrees of freedom and a significance level of α = 0.200 (two-tailed) can be found using a t-table or calculator. The critical value is approximately ±1.94.

Write a function to model the volume of a rectangular prism if the length is 26cm and the sum of the width and height is 32cm. what is the maximum possible volume of the prism?

Answers

To model the volume of a rectangular prism with length 26cm and width w and height h such that the sum of the width and height is 32cm, we can use the following function:

V(w, h) = 26wh

subject to the constraint:

w + h = 32

We can solve for one of the variables in the constraint equation and substitute it into the volume equation, giving us:

w + h = 32  =>  h = 32 - w

V(w) = 26w(32 - w) = 832w - 26w^2

To find the maximum possible volume, we can take the derivative of this function with respect to w and set it equal to zero

dv/dw= 832 - 52w = 0

Solving for w, we get:

w = 16

Substituting this value back into the constraint equation, we get:

h = 32 - w = 16

Therefore, the maximum possible volume of the prism is:

V(16, 16) = 26(16)(16) = 6656 cubic cm

So the function to model the volume of the rectangular prism is V(w) = 832w - 26w^2, and the maximum possible volume is 6656 cubic cm when the width and height are both 16cm.

To Know more about  volume of a rectangular prism refer here

https://brainly.com/question/21308574#

#SPJ11

What is the length of the segment indicated by the question mark

Answers

Check the picture below.

[tex]\begin{array}{llll} \textit{using the pythagorean theorem} \\\\ c^2=a^2+o^2 \end{array} \qquad \begin{cases} c=\stackrel{hypotenuse}{6.6+x}\\ a=\stackrel{adjacent}{6.6}\\ o=\stackrel{opposite}{8.8} \end{cases} \\\\\\ (6.6+x)^2= (6.6)^2 + (8.8)^2\implies (6.6+x)^2=121\implies (6.6+x)^2=11^2 \\\\\\ 6.6+x=11\implies x=4.4[/tex]

Write an expression for the sequence of operations described below.
Subtract three from the product of seven and eight
Type x if you want to use a multiplication sign. Type / if you want to use a division sign. Do not simplify any part of the expression.

Answers

An expression for the sequence of operations described "Subtract three from the product of seven and eight." is (7 × 8) - 3.

How to evaluate and solve the given expression?

In order to evaluate and solve this expression, we would have to apply the PEMDAS rule, where mathematical operations within the parenthesis (grouping symbols) are first of all evaluated, followed by exponent, and then multiplication or division from the left side of the equation to the right. Lastly, the mathematical operations of addition or subtraction would be performed from left to right.

Based on the information provided, we have the following mathematical expression:

Expression: (7 × 8) - 3

56 - 3

53

Read more on expression here: brainly.com/question/16729936

#SPJ1

Edro, Lena, Harriet, and Yermin each plot a point to approximate StartRoot 0. 50 EndRoot.




Pedro A number line going from 0 to 0. 9 in increments of 0. 1. A point is between 0. 2 and 0. 3.



Lena A number line going from 0 to 0. 9 in increments of 0. 1. A point is between 0. 4 and 0. 5.



Harriet A number line going from 0 to 0. 9 in increments of 0. 1. A point is at 0. 5.



Yermin A number line going from 0 to 0. 9 in increments of 0. 1. A point is just to the right of 0. 7.



Whose point is the best approximation of StartRoot 0. 50 EndRoot?


Pedro


Lena


Harriet


Yermin

Answers

Yermin's point is the best approximation of the square root of 0.50.

To know whose point is the best approximation of the square root of 0.50 on a number line. We have the points plotted by Pedro, Lena, Harriet, and Yermin.

Step 1: Calculate the square root of 0.50.
[tex]\sqrt{0.50} = 0.707[/tex]

Step 2: Compare the plotted points to the calculated square root value.
Pedro: Between 0.2 and 0.3
Lena: Between 0.4 and 0.5
Harriet: At 0.5
Yermin: Just to the right of 0.7

Step 3: Determine the closest approximation.
Yermin's point (just to the right of 0.7) is the closest to the calculated value of 0.707.

Your answer: Yermin's point is the best approximation of the square root of 0.50.

To know more about "Square root" refer here:

https://brainly.com/question/29775049#

#SPJ11

Mohal is a waiter at a restaurant. Each day he works, Mohal will make a guaranteed wage of $25, however the additional amount that Mohal earns from tips depends on the number of tables he waits on that day. From past experience, Mohal noticed that he will get about $15 in tips for each table he waits on. How much would Mohal expect to earn in a day on which he waits on 16 tables? How much would Mohal expect to make in a day when waiting on

t tables?

Answers

Answer:

If Mohal waits on 16 tables, he can expect to earn $25 wages + ($15 tips x 16 tables) = $265 in a day.

If Mohal waits on 1 table, he can expect to earn $25 wages + ($15 tips x 1 table) = $40 in a day.

A cake is in the shape of a rectangular prism. It has a length of 13 inches, a width of 8 inches, and a height


of 5 inches. A baker will put frosting on all sides of the cake except for the bottom. What is the total surface area


of the cake that will be covered in frosting?


Show Your Work


O 114 in.


0 334 in.


O 449 in?


O 573 in?

Answers

If the cake is in the shape of a rectangular prism, the total surface area of the cake that will be covered in frosting is 314 sq. inches.

To find the total surface area of the cake that will be covered in frosting, we need to calculate the area of all sides except the bottom. A rectangular prism has 6 sides, and we will be considering 5 of them.

Surface area of top: length × width = 13 × 8 = 104 sq. inches
Surface area of front: length × height = 13 × 5 = 65 sq. inches
Surface area of back: length × height = 13 × 5 = 65 sq. inches
Surface area of left side: width × height = 8 × 5 = 40 sq. inches
Surface area of right side: width × height = 8 × 5 = 40 sq. inches

Now, we will sum the areas of all these sides:
104 + 65 + 65 + 40 + 40 = 314 sq. inches

So, the total surface area of the cake that will be covered in frosting is 314 sq. inches. None of the provided options match this answer, so it is important to double-check the question for any discrepancies.

More on surface area: https://brainly.com/question/10667581

#SPJ11

Malachi ask students in his class, “ how long does it take you to get to school?“ The histogram shows the data

Answers

Answer: C Distribution is symmetric

Step-by-step explanation:

What is the missing value of G if G is two and one-half times smaller than 19. 02 cm? A. 7. 608 cm B. 7. 808 cm C. 8. 608 cm D. 9. 51 cm

Answers

Therefore, the missing value of G is 7.608 cm, which is option A.

What is the missing value of G?

If G is two and one-half times smaller than 19.02 cm, we can find the value of G by multiplying 19.02 cm by 2/5, since two and one-half is equal to five halves, or 2/5 when expressed as a fraction.

G = (2/5) x 19.02 cm

Simplifying this expression:

G = 7.608 cm

Therefore, the missing value of G is 7.608 cm, which is option A.

Learn more about the missing value of G

brainly.com/question/3759414

#SPJ11

Other Questions
Pls help i really need help on this can someone help me pleasehere is the picture is about Row Ops Use a triple integral to find the volume of the solid bounded by the parabolic cylinder y = 2x2 and the planes z = 0,2= 2 and y = 4. A party rental company has chairs and tables for rent. The total cost to rent 3 chairs and 2 tables is $25. Given the measure of an acute angle in a right triangle, we can tell the ratios of the lengths of the triangle's sides relative to that acute angle.Here are the approximate ratios for angle measures 555555, degree, 656565, degree, and 757575, degree.Angle555555, degree656565, degree757575, degreeadjacent leg lengthhypotenuse lengthhypotenuse lengthadjacent leg length start fraction, start text, a, d, j, a, c, e, n, t, space, l, e, g, space, l, e, n, g, t, h, end text, divided by, start text, h, y, p, o, t, e, n, u, s, e, space, l, e, n, g, t, h, end text, end fraction0.570.570, point, 570.420.420, point, 420.260.260, point, 26opposite leg lengthhypotenuse lengthhypotenuse lengthopposite leg length start fraction, start text, o, p, p, o, s, i, t, e, space, l, e, g, space, l, e, n, g, t, h, end text, divided by, start text, h, y, p, o, t, e, n, u, s, e, space, l, e, n, g, t, h, end text, end fraction0.820.820, point, 820.910.910, point, 910.970.970, point, 97opposite leg lengthadjacent leg lengthadjacent leg lengthopposite leg length start fraction, start text, o, p, p, o, s, i, t, e, space, l, e, g, space, l, e, n, g, t, h, end text, divided by, start text, a, d, j, a, c, e, n, t, space, l, e, g, space, l, e, n, g, t, h, end text, end fraction1.431.431, point, 432.142.142, point, 143.733.733, point, 73Use the table to approximate mLmLm, angle, L in the triangle below.3.23.211.911.9LLKKJJChoose 1 answer: Heights for 16-year-old boys are normally distributed with a mean of 68. 3 in. And a standarddeviation of 2. 9 in. Find the z-score associated with the 96th percentile. Find the height of a 16-year-old boy in the 96th percentile. State your answer to the nearest inch If re glazing do you cover what was forgotten(in the 2nd firing) or you freglaze everywhere Abby makes wants to make a gallon of punch. She uses 2 quarts of orange juice 1 cup of lemon juice and 2 1/2 pints of pineapple juice. How many cups of water should you add to make 1 gallon? ASAP!! BRAINLIEST! Please help and show workQuantifying chemical reactions If the arch were 32 inches wide but 44 inches tall high, how could you modify your function W to model the new arch The area of a rectangle is 42 square meters. The length is 7 meters. What is the width?35 meters14 meters15 meters6 meters A vehicle purchased for $29800 depreciates at a constant rate of 7% per year. Determine the approximatevalue of the vehicle 11 years after purchase.Round to the nearest whole number. A model of a helicopter rotor has four blades, each 3.40 m long from the central shaft to the bladetrip. The model is rotated in a wind tunnel at 550 rev/min. (a) What is the linear speed of the bladetip, in m/s? (b) What is the radial acceleration of the blade tip expressed as a multiple of theacceleration of gravity, g? Find an equation in slope-intercept form for the line passing through each pair of points: (4, 7), (1, 4) Compare Jimmy Carter & Ronald Reagan's debate answers to inflation 1. Tina is caring for an infant who is running a very high fever and vomiting whenever Tina tries to feed him some water to keep him hydrated. If she continues to give him water to try to lower his body temperature and increase hydration and, as a result, he vomits more fluid than he is drinking, then he will only get more dehydrated, and his body temperature will rise. What do you think Tina should do to reduce the childs fever, make him more comfortable, and enhance his ability to drink liquids?What are some medical terms that Tina might use when documenting the infant's condition? (at least three) A sample of iodine-131 has an activity of 200 mCi. If the half-life of iodine-131 is 8. 0 days, what activity is observed after 16 days?0. 5 half livesStep [1]: Determine the number of half lives. x 1285. 88 x10-37mci16 days128 half lives2 half lives80 half lives1 dayStep [2]: Find the final activity. 50. 0 mci200. Mci200. Mci(initial activity) A small rubber wheel is used to drive a large pottery wheel. The two wheels are mounted so that their circular edges touch. The small wheel has a radius of 2. 8 cm and accelerates at the rate of 6. 0 rad/s2, and it is in contact with the pottery wheel (radius 23. 0 cm ) without slipping. Calculate (a) the angular acceleration of the pottery wheel, and (b) the time it takes the pottery wheel to reach its required speed of 65 rpm the cfo of the company believes that an appropriate annual interest rate on this investment is 6.5%. what is the present value of this uneven cash flow stream, rounded to the nearest whole dollar? $1,475,000 $467,500 $650,014 $1,692,500 Two observers at point A and B, 150 km apart, sight a balloon between them at angles of elevation 42 and 76 respectively. How far is the observer A from the balloon? Round answer to the nearest tenthPlease show step by step